Sudoku des intégrales convergentes

24

Réponses

  • Je pense qu'on a fait le tour des méthodes existantes.
    J'ai essayé de chercher une primitive de la dite fonction à savoir
    $$ \int \frac {\ln(x)}{(1+x^2)^2} dx,$$ mais je n'ai pas réussi.
    Le 😄 Farceur


  • Bonjour,

    Wolfram sait le faire... avec un polylogarithme... d'où la difficulté de cette intégrale car toutes les manipulations n'ont pour but que d'éliminer cet aspect là de la primitive...
  • Merci YvesM et à un prochain sudoku:-)
    Le 😄 Farceur


  • Bonsoir FDP
    bizarre, je viens de le remarquer !
    je vais le déguster à partir de ce weekend
    Le 😄 Farceur


  • Je soumets la délicieuse intégrale suivante : $$\int_{0}^{\infty} \frac{2 \cos (x) \log x + \pi \sin x}{x^2+4} \ dx $$
  • Avec $f(z)=e^{iz}\log z/(z^2+4)$ et la cle de serrure d'YvesM on devrait y arriver par residus.
  • Bonjour @cyrano et @P.,

    Soit $\displaystyle I = \int_{0}^{+\infty} dx \ {2 \cos x \ln x + \pi \sin x \over x^2+4}.$

    Comme l'intégrande est définie et continue sur $x >0$, et qu'en $x=0$ elle est équivalente à la fonction $\displaystyle x \mapsto \frac12 \ln x$ qui est bien intégrable en $0$, alors $I$ existe.

    On définit la fonction $\displaystyle z \mapsto {e^{iz} \ln z \over z^2 + 4}$ qui est méromorphe sur la surface de Riemann du logarithme privé de $0$ et de $\pm 2i$, deux pôles d'ordre $1.$

    On définit un contour fermé, orienté et parcouru une et une seule fois en demi-cercle supérieur de centre $O$ et de rayon $R>0$, puis le long de l'axe des abscisses entre $-R$ et $+R$ et on n'oublie pas un demi-cercle supérieur de centre $O$ et de rayon $\varepsilon$ pour contourner l'origine car le logarithme n'y est pas défini.

    On calcule alors sans difficulté avec un changment de variable $u=-t$ pour transformer l'intégrale entre $-R$ et $0$ en intégrale entre $0$ et $+R$, avec $\ln(-1) = i\pi$ selon la détermination principale du logarithme, on identifie la partie réelle pour établir : $\displaystyle I = \int_{0}^{+\infty} dx \ {2 \cos x \ln x + \pi \sin x \over x^2+4}= \frac{\pi}{2} e^{-2} \ln 2.$

    Le calcul du résidu en $z=2i$ ne pose pas de problème.

    @P., Le contour en trou de serrure ne marche pas car le terme $\displaystyle e^{iz}$ diverge quand $\displaystyle R \to +\infty$ pour les angles polaires $\theta$ entre $\pi$ et $2\pi.$

    En prime, la partie imaginaire donne le résultat suivant : $\displaystyle \int_{0}^{+\infty} dx \ {\cos x \over x^2+4} = \frac{\pi}{4} e^{-2} .$
  • J'avais suggere un mauvais contour en effet. Bravo Yves.


    PS Respectueusement : quand cesseras tu d'appeler une telle fonction meromorphe sur $\mathbb{C}$? Meromorphe, elle l'est, mais sur la surface de Riemann du logarithme et non sur $\mathbb{C}$.
  • Bonjour,

    @P. : Je dois admettre que je n'écris pas ce qu'il faut sur $z \mapsto \ln z$ quant à son caractère méromorphe parce que, en bon physicien, je n'ai jamais appris cette histoire de surface de Riemann. Je vais essayer de combler cette lacune.

    @Fin de partie,

    Je confirme que cette intégrale est monstrueuse : $I = \displaystyle \int_0^1 \dfrac{x\ln(1-x^2)\arctan x}{1+x^2}\mathrm{d}x$
    mais, en y travaillant pas moins de 20 heures :)o, je confirme le résultat $\displaystyle I =-\dfrac{\pi^3}{48}-\dfrac{\pi}{8} \ln^2 2+ G \ln 2$ avec $G$ la constante de Catalan.

    Seulement, je n'ai pas trouvé le moyen astucieux de simplifier les calculs... ce n'est pas faute d'avoir cherché.

    Pour établir le résultat, sur 18 pages de calcul (écrites à la main), d'abord je transforme les bornes $0$ et $1$ et le $\displaystyle \ln(1-x^2)$ en bornes $-1$ et $+1$ et $\displaystyle \ln (1+x)$ car je suis persuadé qu'il faut abaisser le degré dans le logarithme pour progresser ; puis je passe en complexe par la relation qui gagne à être connue $\displaystyle \arctan \ x = \frac{1}{2i} \ln {1+ i x \over 1-ix}$ pour tout $x$ complexe ; puis j'écris $\displaystyle {x \over 1+x^2} = ({1 \over 1-ix} - {1 \over 1+ix}) \frac{1}{2i}$ et j'obtiens quatre intégrales. Deux sont "faciles" à calculer (1 heure chacune), deux sont "difficiles" avec une vingtaine d'intégrations par parties et changements de variables pour faire apparaître des di- et tri-logarithmes. On rassemble tout ça (10 minutes juste à écrire la formule) et le tour est joué : il ne reste plus qu'à prendre la valeur aux bornes $-1$ et $+1$ (4 heures de travail délicat avec les tables de propritétés des fonctions di- et tri-logarithmes) et voilà !

    Il n'y a aucun intérêt à rédiger cette approche. J'imagine qu'un logiciel formel de pros doit permettre d'y arriver en une ou deux heures de manipulations diverses, mais je n'ai pas accès à une telle merveille.

    Donc je veux bien une méthode raisonnable, si un jour tu la trouves.
  • YvesM:

    20 heures c'est sensiblement le temps qu'il m'avait fallu pour calculer une autre intégrale "monstrueuse" (je ne sais plus si je l'ai indiquée ici). J'avais passé 16 heures non-stop, ou presque, dessus, un travail de dément. B-)-

    Encore merci de l'intérêt porté à cette question.
  • @YvesM
    Ton effort ne doit pas être perdu! j'a cherché sans succès
    Peux tu scanner tes 20 pages de calculs manuscrit et les poster? quand j'aurais le temps, je verrais si on peut simplifier les étapes
    Merci
    Le 😄 Farceur


  • Bonjour,

    Non, mon écriture dans ce genre de recherche n'est pas lisible aux autres.

    Ceci dit, rien est perdu : il suffit de taper dans un logiciel formel les deux intégrales (car dans mon texte, on a bien sûr, quatre intégrales mais deux à deux complexes conjugées) :
    $\displaystyle \int dx \ {\ln (1 - i x) \ln(1-x) \over 1-ix}$ et $\displaystyle \int dx \ {\ln (1 - i x) \ln(1-x) \over 1+ix}$

    Mais la simplification vers le résultat est assez longue...
  • Bonjour
    Ok,

    Amicalement
    Le 😄 Farceur


  • Bonjour,

    Je me demandais si on pouvait exprimer plus simplement l'intégrale:

    $\displaystyle \int_0^\tfrac{\pi}{4}\log(\cos x+\sin x)\log(\cos x)dx$

    (je sais qu'on a: $\cos x+\sin x=\sqrt{2}\cos\left(x-\dfrac{\pi}{4}\right)$ )
  • Salut Fin de Partie,

    Pourquoi ne pas écrire simplement log(cosx+sinx)=log(cosx)+log(1+tanx)?
  • A part ça, l'intégrale généralisée converge.Ne reste plus qu'à calculer explicitement l'intégrale de log(cosx) et de log(1+tanx). Pour la première je suggère un changement de variable u=pi/2-x. Pour la seconde, u=1+tanx ou tanx...
  • En fait, j'ai l'impression que tu es en train de faire le chemin à l'envers du mien. :-D

    L'intégrale que j'ai indiquée plus haut apparait, si je me souviens bien, dans ma tentative d'évaluer l'intégrale:

    $A=\displaystyle \int_0^1 \dfrac{\log(1+x)\arctan(x)}{1+x}$

    En effet,
    Par intégration par parties on a:

    $\displaystyle A=\left[\dfrac{1}{2}(\log(1+x))^2\arctan x\right]_0^1-\dfrac{1}{2} \int_0^1 \dfrac{(\log(1+x))^2}{1+x^2}dx$

    $\displaystyle B= \int_0^1 \dfrac{(\log(1+x))^2}{1+x^2}dx$

    Et on applique le changement de variable $y=\arctan x$ dans cette dernière intégrale.

    $\displaystyle B=\int_0^{\tfrac{\pi}{4}} (\log(1+\tan x))^2dx=\int_0^{\tfrac{\pi}{4}}\Big(\log(\cos x+\sin x)-\log(\cos x)\Big)^2dx$


    Inutile de chercher à calculer
    $\displaystyle \int_0^{\tfrac{\pi}{4}} \log(\cos x)^2 dx$

    C'est une intégrale "horrible".
  • Bonjour
    Peut-être en considèrant la formule \[
    \int_0^x f(t) dt = \sum_{k=0}^{\infty} (-1)^k f^{(k)}(x) \frac{x^{k+1}}{(k+1)!} ,\qquad\qquad (1)
    \] en vérifiant que \[ \lim_{n \to \infty} f^{(n)}(x) \frac{x^{n}}{n!} = 0
    \] Il y a moyen de trouver la solution sous forme d'une série \[
    \int_0^1 f(t) dt = \sum_{k=0}^{\infty} (-1)^k \frac{f^{(k)}(1)}{(k+1)!}
    \] pour le cas de \[ f(x) = \frac{e^{-x} \ln(x) \sin(x)}{\sqrt{x}} \] sans avoir vérifié il me semble que la formule $(1)$ converge vers l'intégrale quand $n$ tend vers l'infini.

    Bonne journée.
  • Je conjecture (je ne sais pas encore le démontrer ) que:

    $\displaystyle \int_0^1 \dfrac{\log(1+x)\arctan(x)}{1+x}dx=-\dfrac{7\pi^3}{128}+\dfrac{\pi(\log 2)^2}{32}+G\log 2+2\Im\left(Li_3\Big(\dfrac{1+i}{2}\Big)\right)$

    $G$ est la constante de Catalan.
  • bonjour, sauf erreur (vu que cela a été fait plus haut) une intégration par partie donne :

    $ I = \int_0^1 \dfrac{\log(1+x) \arctan(x) }{1+x} \mathrm dx \; = \; \frac{1}{2} \log^2(2) \times \frac{\pi}{4} - \frac{1}{2} \int_0^1 \dfrac{ \log^2(1+x)}{1+x^2} \mathrm dx $ et il y a une bonne chance que l'intégrale restante se calcule car Gradshteyn et Ryzhik donnent $ \int_0^1 \dfrac{ \log^2(x)}{1+x^2} \mathrm dx = \pi^2/ 16 $ et sinon, il reste encore la possibilité de développer le dénominateur ou le numérateur en série entière.
    Qui plus est Wolfram alpha sait calculer cette intégrale mais la forme indiquée laisse plutôt penser à un développement en série vu la présence de polylogarithmes:

    https://www.wolframalpha.com/input/?i=integrate+log^2(1+x)/(1+x^2)+dx+from+x=0+to+1

    http://reference.wolfram.com/language/ref/PolyLog.html

    et de la constante de Catalan $ \mathcal{C} $sous la forme:
    $ 1/192 (-4 i (-48 i \log(4) \mathcal{C}-192 Li_3(1/2+i/2)+105 \zeta(3)+4 \log^3(2))+21 \pi^3+12 \pi \log(2) \log(8)+4 i \pi^2 \log(32))$

    On a $ \log^2(1+x) = \displaystyle x^2 \sum_{N=0}^{\infty} (-x)^N \times \sum_{k=0}^N \dfrac{1}{(k+1)(N-k+1)} $ (produit de Cauchy...) résultat qui peut s'arranger.
    A demon  wind propelled me east of the sun
  • Gilles:

    $\displaystyle \int_0^1 \dfrac{(\log(1+x))^2}{1+x^2}dx$

    est une intégrale "horrible" elle s'exprime en fonction de $\Im\left(Li_3\Big(\dfrac{1+i}{2}\Big)\right)$

    Cela dit, pour calculer cette intégrale je pense qu'il faut l'exprimer en fonction de $\displaystyle \int_0^{\tfrac{\pi}{4}}\log(\cos x)^2 dx$ (qui s'exprime en fonction de $\Im\left(Li_3\Big(\dfrac{1+i}{2}\Big)\right)$ )
  • A demon  wind propelled me east of the sun
  • Une autre intégrale dont la valeur (probable) semble indiquer qu'elle doit être calculable par des méthodes élémentaires:

    $\displaystyle \int_0^1 \dfrac{\arctan x \log x}{1+x}dx=\dfrac{G\ln 2}{2}-\dfrac{\pi^3}{64}$

    ($G$ constante de Catalan).
  • Je suis en train d'essayer de calculer l'intégrale

    $I = \displaystyle \int_0^1 \dfrac{x\ln(1-x^2)\arctan x}{1+x^2}\mathrm{d}x$

    par des moyens élémentaires.


    Je l'ai exprimée, si je n'ai pas commis d'erreur, comme une somme d'une dizaine d'intégrales dont je ne sais pas calculer (avec justification) trois d'entre elles que voici:

    $\displaystyle I_1=\int_0^1\dfrac{\arctan y\log(1+y)}{1+y}dy$

    $\displaystyle I_2=\int_0^1 \dfrac{y\arctan y\log y}{1+y^2} dy$

    $\displaystyle I_3=-\int_0^1 \dfrac{\arctan y\log y}{1+y}dy$

    Je connais des expressions (probables) obtenues à l'aide de la fonction lindep de PARI-GP (variante algorithme LLL)
    Les deux premières intégrales sont assez méchantes, la troisième est, en principe, un peu plus gentille.

    Voici ces expressions (probables):

    $\displaystyle I_1=2G\ln(2)-\dfrac{13}{32}\pi(\ln 2)^2-\dfrac{7}{384}\pi^3+2\int_0^{\tfrac{\pi}{4}}(\ln(\cos x))^2dx$


    $\displaystyle I_2=-G\ln(2)+\dfrac{1}{2}\pi(\ln 2)^2-\dfrac{1}{192}\pi^3-2\int_0^{\tfrac{\pi}{4}}(\ln(\cos x))^2dx$


    $I_3=-\dfrac{1}{2}G\ln 2+\dfrac{1}{64}\pi^3$


    L'idée suivie, jusqu'ici, est de trouver des expressions "simplifiées" de $I_1$ et $I_2$ qui seraient combinaisons linéaires des mêmes termes et dont l'un est la méchante intégrale:

    $\displaystyle \int_0^{\tfrac{\pi}{4}}(\ln(\cos x))^2dx$

    sous la forme:

    constante calculée+ intégrale "simple" que je ne sais pas calculer +/-2 $\displaystyle \int_0^{\tfrac{\pi}{4}}(\ln(\cos x))^2dx$

    Pour $I_1$ j'ai obtenu:

    $\displaystyle I_1= \dfrac{7}{32}\pi(\ln 2)^2-\dfrac{1}{4}G\ln 2+2 \int_0^{\tfrac{\pi}{4}}(\ln(\cos x))^2dx-\dfrac{1}{2}\int_0^1 \dfrac{\ln(1+x+x^2+x^3)\ln(1+x^2)}{1+x^2}dx$

    La dernière intégrale écrite peut s'exprimer comme une combinaison linéaire à coefficients rationnels de termes déjà rencontrés ($G\ln 2$ etc) et qui ne sont pas $\displaystyle \int_0^{\tfrac{\pi}{4}}(\ln(\cos x))^2dx$ c'est là tout son intérêt pour moi.
    (je ne sais pas la calculer)

    Pour $I_2$, par intégrations par parties, on peut se ramener au calcul de $\displaystyle \int_0^1(\arctan x)^2dx$, que je sais mener et au calcul de $\displaystyle \int_0^1(\arctan x)^2\ln x dx$ que je ne sais pas faire.


    Même si cela ne réglerait pas tout, je demeure persuadé que $I_3$ doit, au moins, pouvoir se calculer sans sortir la grosse artillerie.

    PS:
    $G$ est la constante de Catalan.
  • Bonjour,

    Je t'aide pour $\displaystyle I_4 = \int_{0}^{1} dx \arctan^2(x) \ln(x)$...

    L'intégrande est continue sur $\displaystyle ]0, 1]$ et est équivalent en $\displaystyle x=0$ à $\displaystyle x^2 \ln(x)$ dont l'intégrale existe en $0$ : donc $\displaystyle I_4$ existe.

    Une intégration par partie donne :
    $\displaystyle I_4 = (x \ln(x) - x) \arctan^2(x) \mid_{0}^{1} - 2 \int_{0}^{1} dx {x \over 1+x^2} \arctan(x) (\ln(x)-1)= \\ \displaystyle = -(\frac{\pi}{4})^2 - 2 I_2 + 2 \int_{0}^{1} dx {x \over 1+x^2} \arctan(x).$ avec ta notation pour $\displaystyle I_2.$

    Pour intégrer $\displaystyle I_5 = \int_{0}^{1} dx {x \over 1+x^2} \arctan(x)$ on écrit d'abord $\displaystyle {x \over 1+x^2} = \frac{1}{2i} ({1 \over 1-ix} - {1 \over 1+ix} )$ puis on sait, ou on fait bien d'apprendre pour se confronter à ces intégrales, que $\displaystyle \arctan(x) = \frac{1}{2i} \ln {1+ix \over 1-ix}$ et on fait appraître la fonction dilogarithme... un peu long à écrire, mais le résultat est :
    $\displaystyle I_5 = \frac{i}{2} (Li_2(-e^{2i\arctan(x)}) + \arctan^2(x) + 2i \arctan(x) \ln(1+e^{2i \arctan(x)})) \mid_{0}^{1} = \\ \displaystyle = (\frac{G}{2} - \frac{\pi}{8}\ln(2) - i\frac{\pi^2}{24}) - (- i\frac{\pi^2}{24}) = \frac{G}{2} - \frac{\pi}{8}\ln(2).$

    On en déduit que $\displaystyle I_4 = -\frac{\pi^2}{16} - 2I_2 + G - \frac{\pi}{4} \ln(2).$
  • Merci. :-)

    Mais ta solution ne remplit pas mon "cahier des charges" (que je sais être idiot).

    Les outils que je m'autorise:

    changement de variable, dérivée sous le signe intégral, calcul sur les séries, intégrales double, triple . Des trucs sans un gramme de nombre complexe.

    (je vais sans doute échouer dans cette entreprise)
  • Bonjour,

    Sans un gramme de complexe... et pour que le défi soit complet : en écoutant de la musique pop togolaise à fond. Voici :

    On veut calculer l'intégrale, qui existe, $\displaystyle I_5 = \int_{0}^{1} dx \ {x \over 1+x^2} \arctan(x).$

    On effectue le changement de variable $\displaystyle x = \tan( t)$ avec $0 < t < \frac{\pi}{2}$ : $\displaystyle I_5 =\int_{0}^{\frac{\pi}{4}} dt \ t \tan(t)$ que l'on intègre par parties, et alors : $\displaystyle I_5 = - t \ln(\cos (t)) \mid_{0}^{\frac{\pi}{4}} + \int_{0}^{\frac{\pi}{4}} dt \ln(\cos (t)) = +\frac{\pi}{8} \ln(2) + \int_{0}^{\frac{\pi}{4}} dt \ln(\cos (t)).$

    On pose $\displaystyle C = \int_{0}^{\frac{\pi}{4}} dt \ln(\cos (t))$ et $\displaystyle S = \int_{0}^{\frac{\pi}{4}} dt \ln(\sin (t)).$ On a alors démontré que $\displaystyle I_5 = +\frac{\pi}{8} \ln(2)+C.$

    On pose $\displaystyle A= \int_{0}^{\frac{\pi}{2}} dt \ln(\cos (t))$ : cette intégrale existe.
    On effectue le changement de variable $\displaystyle y=\frac{\pi}{2}-x$ et alors : $\displaystyle A= \int_{0}^{\frac{\pi}{2}} dt \ln(\sin (t)).$
    Puis on calcule : $\displaystyle A= \int_{0}^{\frac{\pi}{2}} dt \ln(\sin (t)) = \int_{0}^{\frac{\pi}{2}} dt \ln(2\sin (\frac{t}{2}) \cos (\frac{t}{2})) = \frac{\pi}{2} \ln(2) + 2S+2C.$

    On effectue le changement de variable $\displaystyle y=\frac{\pi}{2}-x$ et alors $\displaystyle C = \int_{\frac{\pi}{4}}^{\frac{\pi}{2}} dt \ln(\sin (t))$ puis, enfin, $\displaystyle C +S = \int_{0}^{\frac{\pi}{4}} dt \ln(\sin (t)) + \int_{\frac{\pi}{4}}^{\frac{\pi}{2}} dt \ln(\sin (t)) = \int_{0}^{\frac{\pi}{2}} dt \ln(\sin (t)) = A$ par la relation de Chasles.

    On a démontré que $\displaystyle A = \frac{\pi}{2} \ln(2) + 2(S+C) = \frac{\pi}{2} \ln(2) +2A$ et donc $\displaystyle A = -\frac{\pi}{2} \ln(2).$

    On sait, par définition de la constante de Catalan, notée $G$, que $\displaystyle G = \int_{0}^{\frac{\pi}{4}} dt \ln(\frac{1}{\tan(t)}).$

    Si tu ne reconnais pas cette définition, alors le changement de variable $\displaystyle x = \tan( t)$ avec $0 <t < \frac{\pi}{2}$ donne $\displaystyle G=-\int_{0}^{1} dx {\ln(x) \over 1+x^2} = \sum_{k \geq 0} {(-1)^k \over (2k+1)^2}$ car on développe le dénominateur en série et on intègre term à terme. Sous cette forme de série, la constante de Catalan est définie sans ambiguité sans un gramme de complexe.
    Bien sûr, la définition souvent utilisée $\displaystyle G = \int_{0}^{1} dx \ {\arctan(x) \over x}$ donne cette même série car on part de la relation connue et facile à établir : $\displaystyle \arctan(x) = \sum_{k \geq 0} {(-1)^k x^{2k+1} \over 2k+1}.$


    On a donc $\displaystyle G = C-S.$ Par ailleurs, on a montré que $\displaystyle C+S = -\frac{\pi}{2} \ln(2).$ On peut alors résoudre ce système pour trouver $\displaystyle C = -\frac{\pi}{4} \ln(2) + \frac{G}{2}.$

    Et alors on conclut : $\displaystyle I_5 = +\frac{\pi}{8} \ln(2)+C = -\frac{\pi}{8} \ln(2) + \frac{G}{2}.$
  • Merci de l'intérêt que tu portes à ces questions.

    Pour le calcul de:

    $\displaystyle I_5 = \int_{0}^{1} \dfrac{x }{1+x^2} \arctan xdx$

    On a:

    $\displaystyle I_5=\left[\dfrac{1}{2}\ln(1+x^2)\arctan x\right]_0^1-\dfrac{1}{2}\int_0^1 \dfrac{\ln(1+x^2)}{1+x^2}dx$

    On considère la fonction $f$ définie sur $[0,1]$ par:
    $\displaystyle f(y)=\int_0^1 \dfrac{\ln(1+y^2x^2)}{1+x^2}dx$

    Remarquons que $f(0)=0$.

    On a donc pour tout $y$ de $[0,1]$,
    $\displaystyle f^\prime (y)=\int_0^1 \dfrac{2x^2y}{(1+x^2)(1+x^2y^2)}dx=\left[\dfrac{2y\arctan x}{y^2-1}-\dfrac{2\arctan(xy)}{y^2-1}\right]_0^1=\dfrac{\pi y}{2(y^2-1)}-\dfrac{2\arctan y}{y^2-1}$

    $\displaystyle f^\prime (y)=\dfrac{\pi}{4(1+y)}+\dfrac{\arctan y}{1+y}+\dfrac{4\arctan y-\pi}{4(1-y)}$

    $\displaystyle \int_0^1 \dfrac{\ln(1+x^2)}{1+x^2}dx=f(1)-f(0)=\Big[\dfrac{\pi}{4} \ln(1+x)\Big]_0^1+\left(\Big[\ln(1+x)\arctan x\Big]_0^1-\displaystyle \int_0^1\dfrac{\ln(1+x)}{1+x^2}dx\right)+\\ \left(\Big[-\dfrac{1}{4}\ln(1-x)(4\arctan y-\pi)\Big]_0^1+\displaystyle\int_0^1 \dfrac{\ln(1-x)}{1+x^2}dx\right)=\dfrac{\pi \ln 2}{2}+\displaystyle \int_0^1\dfrac{\ln\left(\tfrac{1-x}{1+x}\right)}{1+x^2}dx$


    Dans la dernière intégrale on fait le changement de variable $y=\dfrac{1-x}{1+x}$,

    $\displaystyle \int_0^1 \dfrac{\ln(1+x^2)}{1+x^2}dx=\dfrac{\pi \ln 2}{2}+\int_0^1 \dfrac{\ln x}{1+x^2}dx$

    cette toute dernière intégrale a pour valeur $-G$ donc:

    $\displaystyle \int_0^1 \dfrac{\ln(1+x^2)}{1+x^2}dx=\dfrac{\pi \ln 2}{2}-G$

    et ainsi,

    $I_5=\dfrac{G}{2}-\dfrac{\pi}{8}\ln 2$


    Il me reste à voir si ce dernier calcul permet effectivement de calculer par des méthodes élémentaires l'intégrale qui me préoccupe.
  • La tâche est rude.

    En bricolant un peu, j'ai trouvé une relation (que je sais démontrer) entre $I_2$ et $I_3$ non triviale.


    $\displaystyle I_2=-\dfrac{G\ln 2}{2}-\dfrac{\pi^3}{96}+\dfrac{\pi}{4}(\ln 2)^2-\int_0^{\tfrac{\pi}{4}}(\ln(\cos x))^2dx-\dfrac{1}{2}\int_0^1\dfrac{\arctan x \ln x}{1-x}dx-\dfrac{1}{2}I_3$


    Cela permet de "sortir" $-\dfrac{G\ln 2}{2}$ et $\displaystyle -\int_0^{\tfrac{\pi}{4}}(\ln(\cos x))^2dx$
    mais c'est insuffisant.


    J'ai raisonné à l'envers. Comment construire une fonction (pas trop stupide) dont l'intégrale est $-G\ln 2$?

    Je suis parti de l'égalité toute "bête":

    $\Big[\ln(1+x^2)T(x)\Big]_0^1=-G\ln 2$

    Où, $\displaystyle T(x)=\int_0^x \dfrac{\ln t}{1+t^2}dt$ avec $x>0$ (on a $T(1)=-G$)

    On peut procéder de la même façon avec $\ln(1+x)T(x)$ pour obtenir une autre relation entre $I_2$ et $I_3$ avec deux intégrales parasites.

    PS:
    Malheureusement le calcul de $I_5$ ne fait pas avancer la résolution du problème. :(

    En effet,

    $\displaystyle I_2=-\dfrac{1}{2}\int_0^1 (\arctan x)^2\ln x dx-\dfrac{1}{2}\int_0^1 (\arctan x)^2 dx$

    (j'ai considéré qu'une primitive de $\dfrac{\arctan x}{1+x^2}$ est $\dfrac{1}{2}(\arctan x)^2$)

    (en remplaçant la première intégrale par son expression en fonction de $I_2$ indiquée plus haut par YvesM on va se retrouver avec un terme en $I_2$ exactement :( )
    Mais cela donne un moyen de calculer $\displaystyle \int_0^1 (\arctan x)^2 dx$. :-D
  • La deuxième relation entre $I_2$ et $I_3$ que je sais démontrer:

    $\displaystyle I_2=2G\ln 2-\dfrac{\pi^3}{48}-\dfrac{\pi(\ln 2)^2}{4}+\int_0^{\tfrac{\pi}{4}}(\ln(\cos x))^2 dx+\int_0^1 \dfrac{\ln(1+x)\ln x}{1+x^2}-I_3$

    Je suis parti, comme indiqué plus haut de l'égalité $\Big [\ln(1+x)T(x)\Big]_0^1=-G\ln 2$

    PS:
    Cela permet de donner une autre expression de $I_3$ mais qui ne me semble pas plus simple à traiter.
  • Une égalité que je trouve jolie:

    $\displaystyle \int_0^{\tfrac{\pi}{4}}\ln(\cos x-\sin x)\ln(\cos x) dx - \int_0^{\tfrac{\pi}{4}}\ln(\cos x+\sin x)\ln(\sin x)dx=\dfrac{G\ln 2}{2}$
  • Je pense que j'ai réussi à démontrer, par des méthodes élémentaires, que:

    $\displaystyle \int_0^1 \dfrac{\arctan x \log x}{1+x}dx=\dfrac{G\ln 2}{2}-\dfrac{\pi^3}{64}$



    On définit pour $x\in [0,1]$ la fonction $F$:

    $\displaystyle F(x)=\int_0^x \dfrac{\log t}{1+t}dt$

    Noter que $F(1)=-\dfrac{\pi^2}{12}$

    (utiliser un développement en série de Taylor)

    et, après le changement de variable, $y=\dfrac{t}{x}$, on a:

    $\displaystyle F(x)=\int_0^1 \dfrac{x\log(xy)}{1+xy}dy$

    Puisque:

    $\Big[F(x)\arctan x\Big]_0^1=-\dfrac{\pi^3}{48}$

    alors,

    $\displaystyle -\dfrac{\pi^3}{48}=\int_0^1 \dfrac{F(x)}{1+x^2}dx+\int_0^1 \dfrac{\arctan x\log x}{1+x}dx$

    $\displaystyle\int_0^1 \dfrac{F(x)}{1+x^2}dx=\int_0^1 \dfrac{x\log(xy)}{(1+xy)(1+x^2)}dxdy$

    $\displaystyle\int_0^1 \dfrac{F(x)}{1+x^2}dx=\int_0^1 \dfrac{x\log(x)}{(1+xy)(1+x^2)}dxdy+\int_0^1 \dfrac{x\log(y)}{(1+xy)(1+x^2)}dxdy$

    $\displaystyle\int_0^1 \dfrac{F(x)}{1+x^2}dx=\int_0^1\left[\dfrac{\log x\log(1+xy)}{1+x^2}\right]_{y=0}^{y=1} dx+
    \displaystyle \int_0^1 \left[-\dfrac{\log y\log(1+xy)}{1+y^2}+\dfrac{\log y\log(1+x^2)}{2(1+y^2)}+\dfrac{y\log y\arctan x}{1+y^2}\right]_{x=0}^{x=1}dy$


    $\displaystyle\int_0^1 \dfrac{F(x)}{1+x^2}dx= \int_0^1 \dfrac{\log x\log(1+x)}{1+x^2}dx-\int_0^1\dfrac{\log y\log(1+y)}{1+y^2}dy+\dfrac{\log 2}{2}\int_0^1 \dfrac{\log y}{1+y^2}dy+
    \dfrac{\pi}{4}\times \int_0^1 \dfrac{y\log y}{1+y^2}dy$

    en utilisant un développement en série de Taylor on obtient:

    $\displaystyle \int_0^1 \dfrac{y\log y}{1+y^2}dy=-\dfrac{\pi^2}{48}$
    Et il est bien connu que , $\displaystyle -G=\int_0^1\dfrac{\log y}{1+y^2}dy$

    Donc,

    $\displaystyle\int_0^1 \dfrac{F(x)}{1+x^2}dx=-\dfrac{G\log 2}{2}-\dfrac{\pi^3}{192}$

    Et finalement,

    $\displaystyle \int_0^1 \dfrac{\arctan x \log x}{1+x}dx=\dfrac{G\ln 2}{2}-\dfrac{\pi^3}{64}$

    En espérant n'avoir pas écrit trop d'énormités.
  • Le plus dur reste probablement à faire.

    En considérant la fonction $\displaystyle F(x)=\int_0^x \dfrac{t\ln t}{1+t^2}dt$

    Je parviens à montrer que:

    $\displaystyle I_2=-\dfrac{2}{3}\int_0^1 \dfrac{\ln x\ln(1+x^2) }{1+x^2}dx-\dfrac{1}{3}\int_0^1 \dfrac{\ln x\arctan x}{1-x}dx-\dfrac{1}{2}G\ln 2$

    (Ce qui permet, très probablement, compte tenu de ce qui figure précédemment de trouver une relation entre les deux intégrales dans le membre de droite)

    PS:
    $\displaystyle \int_0^1 \dfrac{\ln x\ln(1+x^2) }{1+x^2}dx$ pourrait se montrer très importante car, sauf erreur, on sait l'écrire comme une combinaison linéaire des constantes déjà rencontrées et de l'intégrale embarrassante $\displaystyle \int_0^{\tfrac{\pi}{4}} \ln(\cos x)^2 dx$
  • De http://www.les-mathematiques.net/phorum/read.php?4,1236063,1293269#msg-1293269

    On obtient:

    $\displaystyle \int_0^1\dfrac{\arctan x \ln x}{1-x}dx=-2\int_0^1 \dfrac{\ln x\ln(1+x^2) }{1+x^2}dx-\dfrac{3}{2}G\ln 2-3I_2$

    De http://www.les-mathematiques.net/phorum/read.php?4,1236063,1292237#msg-1292237

    et

    http://www.les-mathematiques.net/phorum/read.php?4,1236063,1292791#msg-1292791

    On déduit:

    $\displaystyle I_2=-G\ln 2+\dfrac{7\pi^3}{192}-\dfrac{\pi}{2}(\ln 2)^2+2\int_0^{\tfrac{\pi}{4}}(\ln(\cos x))^2dx-2\int_0^1\dfrac{\ln x\ln(1+x^2)}{1+x^2}dx$

    En procédant au changement de variable $y=\tan x$ on obtient:

    $\displaystyle\int_0^1\dfrac{\ln x\ln(1+x^2)}{1+x^2}dx=\int_0^{\tfrac{\pi}{4}} \ln(\tan x)\ln\left(\dfrac{1}{(\cos x)^2}\right)dx= 2\int_0^{\tfrac{\pi}{4}}(\ln(\cos x))^2dx-2\int_0^{\tfrac{\pi}{4}}\ln(\sin x)\ln(\cos x)dx$

    Sachant que $\displaystyle \sum_{n=0}^{+\infty} \dfrac{(-1)^n}{(2n+1)^3}=\dfrac{\pi^3}{32}$ (ce n'est pas un résultat trivial)


    Je peux démontrer que:
    $\displaystyle \int_0^{\tfrac{\pi}{4}}\ln(\sin x)\ln(\cos x)dx=\dfrac{\pi}{4}(\ln 2)^2-\dfrac{\pi^3}{96}$

    et ainsi,

    $\displaystyle I_2=-G\ln 2-\dfrac{\pi^3}{192}+\dfrac{\pi}{2}(\ln 2)^2-2\int_0^{\tfrac{\pi}{4}}(\ln(\cos x))^2dx$
  • Un intermède,

    Montrer que $\displaystyle \int_0^1 \dfrac{\ln x\ln(1+x^2)}{1-x}dx=2\zeta(3)-\dfrac{3}{16}\pi^2\ln 2-\dfrac{1}{2}G\pi$
  • Un intermède de plus,

    Montrer que,
    $\displaystyle \int_0^1 \dfrac{\ln x\ln(1+x^2)}{1+x}dx=\dfrac{3}{2}\zeta(3)-\dfrac{1}{16}\pi^2\ln 2-\dfrac{1}{2}G\pi$
  • Bonjour à tous , ces intégrales sont assez terrifiantes ::o(où avez vous assez de courage pour les dissoudre ?) j'en propose humblement une :53997
  • Max a écrit:
    Bonjour à tous , ces intégrales sont assez terrifiantes

    Elles sont faussement terrifiantes mais je n'en dis pas plus pour le moment pour ne pas vous enlever le plaisir de chercher les résultats annoncés. (j'ai une preuve pour chacune des intégrales que j'ai proposées) B-)-

    J'ai un autre résultat assez curieux, selon moi, que j'ai découvert par hasard en vérifiant un calcul. J'en parlerai plus tard.
  • pour la derniere, voir Ilan Nardi American Mathematical Monthly avril 1988.page 314.
  • $\displaystyle J=\int_0^1 \dfrac{\ln\left(\ln\left(\dfrac{1}{x}\right)\right)}{1+x^2}dx$

    On applique le changement de variable $y=\dfrac{1}{x}$ il vient,

    $\displaystyle J=\int_1^{+\infty} \dfrac{\ln\left(\ln x\right)}{1+x^2}dx$

    On applique le changement de variable $y=\ln x$ il vient,

    $\displaystyle J=\int_0^{+\infty} \dfrac{e^{x}\ln x}{1+e^{2x}}dx=\int_0^{+\infty} \dfrac{e^{-x}\ln x}{1+e^{-2x}}dx$

    On définit $\displaystyle F(s)=\int_0^{+\infty} \dfrac{e^{-x}x^s}{1+e^{-2x}}dx$ pour $s\geq 0$.

    $\displaystyle F(s)=\int_0^{+\infty}\left(\sum_{n=0}^{+\infty} x^s(-1)^n e^{-(2n+1)x} \right)dx$

    $\displaystyle F(s)=\sum_{n=0}^{+\infty}(-1)^n\left( \int_0^{+\infty} x^s e^{-(2n+1)x} dx\right)$

    Dans cette dernière intégrale on applique le changement de variable $y=(2n+1)x$

    $\displaystyle F(s)=\left(\int_0^{+\infty} x^se^{-x}dx\right)\left(\sum_{n=0}^{+\infty}\dfrac{(-1)^n}{(2n+1)^{s+1}}\right)$

    $\displaystyle F(s)=\Gamma(s+1)\beta(s+1)$

    Où $\beta$ est la fonction beta de Dirichlet.

    $\displaystyle F^\prime(s)=\int_0^{+\infty} \dfrac{x^se^{-x}\ln x}{1+e^{-2x}}dx$

    On a $F^\prime(0)=J$

    $F^\prime(s)=\Gamma^\prime(s+1)\beta(s+1)+\Gamma(s+1)\beta^\prime (s+1)$

    $F^\prime(0)=\Gamma^\prime(1)\beta(1)+\Gamma(1)\beta^\prime (1)=-\gamma\dfrac{\pi}{4}+\dfrac{\pi}{4}\left(\gamma +2\ln 2+3\ln \pi-4\ln\left(\Gamma\left(\dfrac{1}{4}\right)\right)\right)$

    $F^\prime(0)=\dfrac{\pi}{2}\left(\ln 2+\dfrac{3}{2}\ln \pi-2\ln\left(\Gamma\left(\dfrac{1}{4}\right)\right)\right)$

    La formule des compléments donne:

    $\ln\left(\Gamma\left(\dfrac{1}{4}\right)\right)+\ln\left(\Gamma\left(\dfrac{3}{4}\right)\right)=\ln(\pi)+\dfrac{1}{2}\ln 2$

    $F^\prime(0)=\dfrac{\pi}{2}\left(\ln\left(\Gamma\left(\dfrac{1}{4}\right)\right)+\ln\left(\Gamma\left(\dfrac{3}{4}\right)\right)+\dfrac{1}{2}\ln 2+\dfrac{1}{2}\ln \pi-2\ln\left(\Gamma\left(\dfrac{1}{4}\right)\right)\right)$

    $F^\prime(0)=\dfrac{\pi}{2}\left(\ln\left(\Gamma\left(\dfrac{3}{4}\right)\right)+\dfrac{1}{2}\ln 2+\dfrac{1}{2}\ln \pi-
    \ln\left(\Gamma\left(\dfrac{1}{4}\right)\right)\right)$

    $J=F^\prime(0)=\dfrac{\pi}{2}\ln\left(\sqrt{2\pi}\dfrac{\Gamma\left(\dfrac{3}{4}\right)}{\Gamma\left(\dfrac{1}{4}\right)}\right)$



    Voir ici pour les valeurs de $\Gamma^\prime(1),\beta^\prime(1)$
    http://mathworld.wolfram.com/DirichletBetaFunction.html
    http://mathworld.wolfram.com/GammaFunction.html
  • J'aime bien ces deux-là aussi,

    $\displaystyle \int_0^1 \dfrac{\ln x\ln(1+x)}{1-x}dx=\zeta(3)-\dfrac{1}{4}\pi^2\ln 2$

    et son inséparable grande soeur,

    $\displaystyle \int_0^1 \dfrac{\ln x\ln(1+x)}{1+x}dx=-\dfrac{1}{8}\zeta(3)$

    (je n'ai pas obtenu toutes ces formules par une méthode empirique, j'en ai des preuves simples)


    PS:
    J'ai parcouru l'article cité par P., que je remercie pour cette référence.
    La méthode est la même, si je lis bien, que celle que j'ai utilisée. Mais il y a une différence de taille. Ilan Nardi calcule $\Gamma^\prime(1),\beta^\prime(1)$ :-)
  • Si tu aimes FDP la fonction zêta alors en voici une que tu devrais apprécier :-D54011
  • Merci pour cette intégrale.
    Que signifie le surlignage du nombre 5 dans $\zeta(\overline{5},1,1)$?

    PS:
    Ok, je sais ce que c'est. B-)-
  • Fin de partie écrivait : http://www.les-mathematiques.net/phorum/read.php?4,1236063,1299593#msg-1299593
    > $J=F^\prime(0)=\dfrac{\pi}{2}\ln\left(\sqrt{2\pi}\dfrac{\Gamma\left(\dfrac{3}{4}\right)}{\Gamma\left(\dfrac{1}{4}\right)}\right)$

    Avec la formule de duplication de la fonction gamma, tu vas pouvoir simplifier un peu (virer un $\Gamma(\frac34)$)
    Bien vu en tout cas ;)
  • J'ai présenté la solution comme elle était demandée dans la question.

    Quand tu as une évaluation de la fonction $\gamma$ en deux nombres dont la somme est $1$ c'est un bon réflexe de penser à cette formule de duplication. B-)-

    Cela dit , la solution proposée plus haut est une simple esquisse: aucune interversion somme/intégrale n'est justifiée et les parties les plus difficiles du calcul, i.e le calcul de $\beta^\prime(1),\Gamma^\prime(1)$, ne sont pas faites.

    J'espère qu'il y a une autre méthode plus simple pour calculer cette intégrale.
  • Une petite dernière pour la route



    PS:FDP comme je suis nouveau une petite question d'où viens ta passion pour Samuel Beckett ?54041
  • Max8188 a écrit:
    d'où viens ta passion pour Samuel Beckett ?

    Ce pseudo a au moins 4 sens. La référence à Beckett en est un. La référence au jeu d'échec en est un autre.
    Les initiales FDP en sont un autre. B-)-
  • Voici une bizarrerie : $$ \frac{\pi}{2}\ln\bigg(\frac{\int_{0}^{\pi} {\sqrt{\sin(x)} }}{\sqrt 8} dx\bigg)=\int_{0}^{1}\frac{\ln\big(\ln(1/x)\big)}{1+x^{2}}dx$$


    (Merci à l'administrateur qui corrige mes divagations)
  • Numériquement il n'y a pas égalité si je vois bien.
Connectez-vous ou Inscrivez-vous pour répondre.